LSAT and Law School Admissions Forum

Get expert LSAT preparation and law school admissions advice from PowerScore Test Preparation.

General questions relating to the LSAT or LSAT preparation.
User avatar
 lsatquestions
  • Posts: 66
  • Joined: Nov 08, 2021
|
#96709
Hi,

Can you please explain the Justify Formula vs. Mechanistic Approach? When should I be applying one formula over the other for justify (sufficient assumption) questions?
User avatar
 atierney
PowerScore Staff
  • PowerScore Staff
  • Posts: 215
  • Joined: Jul 06, 2021
|
#96761
Yes, the short answer is that the Justify Formula is really just an overview of what each justify question is asking you to do, mainly to identify that answer choice that, when combined with the premises stated in the stimulus allows the conclusion to be properly drawn. The Mechanistic Approach, on the other hand, is a way to help you to break down specifics of each kind of justify question, which will differ in terms of what information is present in the conclusion and premises of the stimulus.

My advice to you is to understand both, and in doing so, utilize this understanding to help you with the Justify questions. The specific question you're asking here is a bit odd in this regard, because I wouldn't view the two as mutually exclusive in any way, or that the use of either in some way renders the other inapplicable. To draw an analogy, it would be like asking whether you should use arithmetic or completing the square when trying to solve a quadratic equation in algebra. You would probably need to use both, although completing is particular method that in now way interferes with whatever necessary addition or subtraction you might need to employ to solve the equation.

Let me know if you have further questions on this.

Get the most out of your LSAT Prep Plus subscription.

Analyze and track your performance with our Testing and Analytics Package.